Home / IBDP Maths AHL 3.11 Relationships in trigonometric functions AA HL Paper 1- Exam Style Questions

IBDP Maths AHL 3.11 Relationships in trigonometric functions AA HL Paper 1- Exam Style Questions

IBDP Maths AHL 3.11 Relationships in trigonometric functions AA HL Paper 1- Exam Style Questions- New Syllabus

Question

The function \( f \) is defined by \( f(x) = \sin(qx) \), where \( q > 0 \). The following diagram shows part of the graph of \( f \) for \( 0 \leq x \leq 4m \), where \( x \) is in radians. There are x-intercepts at \( x = 0, 2m \) and \( 4m \).

Graph of sin(qx)

(a) Find an expression for \( m \) in terms of \( q \).

The function \( g \) is defined by \( g(x) = 3\sin\left(\frac{2qx}{3}\right) \), for \( 0 \leq x \leq 6m \).

(b) On the axes above, sketch the graph of \( g \).

▶️ Answer/Explanation
Solution (a)

The period \( T \) of \( f(x) = \sin(qx) \) is:

\( T = \frac{2\pi}{q} \)

Given the x-intercepts at \( x = 0, 2m, 4m \), we observe that \( 4m \) represents one full period:

\( 4m = \frac{2\pi}{q} \)

Solving for \( m \):

\( m = \frac{2\pi}{4q} = \frac{\pi}{2q} \)

✅ Final Answer:
\( m = \frac{\pi}{2q} \)
Solution (b)

For \( g(x) = 3\sin\left(\frac{2qx}{3}\right) \):

Amplitude = 3

Period \( T = \frac{2\pi}{\frac{2q}{3}} = \frac{3\pi}{q} = 3m \) (using part (a) result)

Key points for one period (0 to \( 3m \)):

  • Start at (0,0)
  • Maximum at \( \left(\frac{3m}{4}, 3\right) \)
  • Midpoint at \( \left(\frac{3m}{2}, 0\right) \)
  • Minimum at \( \left(\frac{9m}{4}, -3\right) \)
  • End at \( (3m,0) \)

Repeat this pattern for the interval \( 3m \) to \( 6m \).

The graph should show:

  • X-intercepts at \( x = 0, 1.5m, 3m, 4.5m, 6m \)
  • Maxima at \( x = 0.75m \) and \( 3.75m \)
  • Minima at \( x = 2.25m \) and \( 5.25m \)
Graph of 3sin(2qx/3)
Question

The angle \( \theta \) satisfies the equation \( \tan\theta + \cot\theta = 3 \). Find all the possible values of \( \theta \) in \( [0^\circ, 90^\circ] \).

▶️ Answer/Explanation
Method 1

Start with the given equation:

\( \tan\theta + \cot\theta = 3 \)

Express \( \cot\theta \) as \( \frac{1}{\tan\theta} \):

\( \tan\theta + \frac{1}{\tan\theta} = 3 \)

Multiply through by \( \tan\theta \):

\( \tan^2\theta – 3\tan\theta + 1 = 0 \)

Solve the quadratic equation:

\( \tan\theta = \frac{3 \pm \sqrt{9 – 4}}{2} = \frac{3 \pm \sqrt{5}}{2} \)

Calculate approximate values:

\( \tan\theta = 0.382 \) or \( 2.618 \)

Find corresponding angles:

\( \theta = \tan^{-1}(0.382) \approx 20.9^\circ \)

\( \theta = \tan^{-1}(2.618) \approx 69.1^\circ \)

Method 2

Express in terms of sine and cosine:

\( \frac{\sin\theta}{\cos\theta} + \frac{\cos\theta}{\sin\theta} = 3 \)

Combine fractions:

\( \frac{\sin^2\theta + \cos^2\theta}{\sin\theta\cos\theta} = 3 \)

Simplify numerator:

\( \frac{1}{\sin\theta\cos\theta} = 3 \)

Use double-angle identity:

\( \frac{2}{\sin 2\theta} = 3 \) ⇒ \( \sin 2\theta = \frac{2}{3} \)

Solve for \( \theta \):

\( 2\theta = \sin^{-1}\left(\frac{2}{3}\right) \approx 41.81^\circ \) or \( 138.19^\circ \)

Thus:

\( \theta \approx 20.9^\circ \) or \( 69.1^\circ \)

✅ Final Answer:
\( \theta \approx 20.9^\circ \) and \( \theta \approx 69.1^\circ \)
Question

(a) Sketch the graph of \( y = \cos(4x) \), in the interval \( 0 \leq x \leq \pi \).

(b) On the same diagram sketch the graph of \( y = \sec(4x) \), for \( 0 \leq x \leq \pi \), indicating clearly the equations of any asymptotes.

(c) Use your sketch to solve:

(i) The equation \( \sec(4x) = -1 \), for \( 0 \leq x \leq \pi \).

(ii) The inequality \( \cos(4x) \leq 0 \), for \( 0 \leq x \leq \pi \).

▶️ Answer/Explanation
Graphical Solution
Graph of cos(4x) and sec(4x)

Key features of \( y = \cos(4x) \):

  • Period: \( \frac{2\pi}{4} = \frac{\pi}{2} \)
  • Amplitude: 1
  • Zeros at \( x = \frac{\pi}{8}, \frac{3\pi}{8}, \frac{5\pi}{8}, \frac{7\pi}{8} \)

Key features of \( y = \sec(4x) \):

  • Vertical asymptotes where \( \cos(4x) = 0 \): \( x = \frac{\pi}{8} + \frac{k\pi}{4} \), \( k \in \mathbb{Z} \)
  • Range: \( (-\infty, -1] \cup [1, \infty) \)
  • Reciprocal of \( \cos(4x) \), so undefined at zeros of cosine
Solution (c)(i)

Solve \( \sec(4x) = -1 \):

This occurs when \( \cos(4x) = -1 \).

Within \( 0 \leq x \leq \pi \), solutions are:

\( 4x = \pi \) ⇒ \( x = \frac{\pi}{4} \)

\( 4x = 3\pi \) ⇒ \( x = \frac{3\pi}{4} \)

Solutions: \( x = \frac{\pi}{4}, \frac{3\pi}{4} \)
Solution (c)(ii)

Solve \( \cos(4x) \leq 0 \):

From the graph, this occurs in the intervals where cosine is negative:

\( \frac{\pi}{8} \leq x \leq \frac{3\pi}{8} \) and \( \frac{5\pi}{8} \leq x \leq \frac{7\pi}{8} \)

Solution: \( x \in \left[\frac{\pi}{8}, \frac{3\pi}{8}\right] \cup \left[\frac{5\pi}{8}, \frac{7\pi}{8}\right] \)
Question

The diagram below shows the boundary of the cross-section of a water channel.

Water channel cross-section

The equation that represents this boundary is \( y = 16\sec\left(\frac{\pi x}{36}\right) – 32 \) where \( x \) and \( y \) are both measured in cm.

The top of the channel is level with the ground and has a width of 24 cm. The maximum depth of the channel is 16 cm.

Find the width of the water surface in the channel when the water depth is 10 cm. Give your answer in the form \( a \arccos b \) where \( a, b \in \mathbb{R} \).

▶️ Answer/Explanation

Solution:

For a water depth of 10 cm, set \( y = -10 \) (since depth is measured downward):

\( -10 = 16\sec\left(\frac{\pi x}{36}\right) – 32 \)

Rearrange the equation:

\( 16\sec\left(\frac{\pi x}{36}\right) = 22 \)

\( \sec\left(\frac{\pi x}{36}\right) = \frac{22}{16} = \frac{11}{8} \)

Convert to cosine:

\( \cos\left(\frac{\pi x}{36}\right) = \frac{8}{11} \)

Solve for \( x \):

\( \frac{\pi x}{36} = \pm \arccos\left(\frac{8}{11}\right) \)

\( x = \pm \frac{36}{\pi} \arccos\left(\frac{8}{11}\right) \)

The total width is the distance between the two x-values:

\( \text{Width} = \frac{36}{\pi} \arccos\left(\frac{8}{11}\right) – \left(-\frac{36}{\pi} \arccos\left(\frac{8}{11}\right)\right) \)

\( \text{Width} = \frac{72}{\pi} \arccos\left(\frac{8}{11}\right) \) cm

✅ Final Answer:
\( \frac{72}{\pi} \arccos\left(\frac{8}{11}\right) \) cm
Scroll to Top